Eigenvalues of a real orthogonal matrix. The Next CEO of Stack OverflowDo real matrices always have real eigenvalues?Generalized eigenvalue problem; why do real eigenvalues exist?If $A$ is a real symmetric matrix, then $A$ has real eigenvalues.Block diagonal form of elements of SO(n)Eigenvectors and eigenvalues of Hessian matrixproperties of, 3x3 matrix, determinant 1, real eigenvaluesWhy eigenvalues of an orthogonal matrix made with QR decomposition include -1?Determine the matrix of the orthogonal projectionLet $A in mathbbC^n times n$ be hermitian. Prove all eigenvalues of $A$ are real…Existence condition of Real Eigenvalues for Non-Symmetric Real Matrix

Would a grinding machine be a simple and workable propulsion system for an interplanetary spacecraft?

pgfplots: How to draw a tangent graph below two others?

My ex-girlfriend uses my Apple ID to login to her iPad, do I have to give her my Apple ID password to reset it?

Identify and count spells (Distinctive events within each group)

Direct Implications Between USA and UK in Event of No-Deal Brexit

Read/write a pipe-delimited file line by line with some simple text manipulation

How can I separate the number from the unit in argument?

How can I prove that a state of equilibrium is unstable?

Calculating discount not working

Is the 21st century's idea of "freedom of speech" based on precedent?

Can this transistor (2N2222) take 6 V on emitter-base? Am I reading the datasheet incorrectly?

Arrows in tikz Markov chain diagram overlap

Small nick on power cord from an electric alarm clock, and copper wiring exposed but intact

Does the Idaho Potato Commission associate potato skins with healthy eating?

Could you use a laser beam as a modulated carrier wave for radio signal?

Is it correct to say moon starry nights?

What did the word "leisure" mean in late 18th Century usage?

What happens if you break a law in another country outside of that country?

Is a linearly independent set whose span is dense a Schauder basis?

Why was Sir Cadogan fired?

Find a path from s to t using as few red nodes as possible

Is it OK to decorate a log book cover?

Free fall ellipse or parabola?

Does Germany produce more waste than the US?



Eigenvalues of a real orthogonal matrix.



The Next CEO of Stack OverflowDo real matrices always have real eigenvalues?Generalized eigenvalue problem; why do real eigenvalues exist?If $A$ is a real symmetric matrix, then $A$ has real eigenvalues.Block diagonal form of elements of SO(n)Eigenvectors and eigenvalues of Hessian matrixproperties of, 3x3 matrix, determinant 1, real eigenvaluesWhy eigenvalues of an orthogonal matrix made with QR decomposition include -1?Determine the matrix of the orthogonal projectionLet $A in mathbbC^n times n$ be hermitian. Prove all eigenvalues of $A$ are real…Existence condition of Real Eigenvalues for Non-Symmetric Real Matrix










4












$begingroup$


Let $A$ be a real orthogonal matrix. Then $A^text T A = I.$ Let $lambda in Bbb C$ be an eigenvalue of $A$ corresponding to the eigenvector $X in Bbb C^n.$ Then we have





$$beginalign* X^text T A^text T A X = X^text T X. \ implies (AX)^text T AX & = X^text T X. \ implies (lambda X)^text T lambda X & = X^text T X. \ implies lambda^2 X^text T X & = X^text T X. \ implies (lambda^2 - 1) X^text T X & = 0. endalign*$$





Since $X$ is an eigenvector $X neq 0.$ Therefore $_2^2 = X^text T X neq 0.$ Hence we must have $lambda^2 - 1 = 0$ i.e. $lambda^2 = 1.$ So $lambda = pm 1.$



So according to my argument above it follows that eigenvalues of a real orthogonal matrix are $pm 1.$ But I think that I am wrong as I know that the eigenvalues of an orthogonal matrix are unit modulus i.e. they lie on the unit circle.



What's going wrong in my argument above. Please help me in this regard.



Thank you very much for your valuable time.










share|cite|improve this question











$endgroup$
















    4












    $begingroup$


    Let $A$ be a real orthogonal matrix. Then $A^text T A = I.$ Let $lambda in Bbb C$ be an eigenvalue of $A$ corresponding to the eigenvector $X in Bbb C^n.$ Then we have





    $$beginalign* X^text T A^text T A X = X^text T X. \ implies (AX)^text T AX & = X^text T X. \ implies (lambda X)^text T lambda X & = X^text T X. \ implies lambda^2 X^text T X & = X^text T X. \ implies (lambda^2 - 1) X^text T X & = 0. endalign*$$





    Since $X$ is an eigenvector $X neq 0.$ Therefore $_2^2 = X^text T X neq 0.$ Hence we must have $lambda^2 - 1 = 0$ i.e. $lambda^2 = 1.$ So $lambda = pm 1.$



    So according to my argument above it follows that eigenvalues of a real orthogonal matrix are $pm 1.$ But I think that I am wrong as I know that the eigenvalues of an orthogonal matrix are unit modulus i.e. they lie on the unit circle.



    What's going wrong in my argument above. Please help me in this regard.



    Thank you very much for your valuable time.










    share|cite|improve this question











    $endgroup$














      4












      4








      4


      0



      $begingroup$


      Let $A$ be a real orthogonal matrix. Then $A^text T A = I.$ Let $lambda in Bbb C$ be an eigenvalue of $A$ corresponding to the eigenvector $X in Bbb C^n.$ Then we have





      $$beginalign* X^text T A^text T A X = X^text T X. \ implies (AX)^text T AX & = X^text T X. \ implies (lambda X)^text T lambda X & = X^text T X. \ implies lambda^2 X^text T X & = X^text T X. \ implies (lambda^2 - 1) X^text T X & = 0. endalign*$$





      Since $X$ is an eigenvector $X neq 0.$ Therefore $_2^2 = X^text T X neq 0.$ Hence we must have $lambda^2 - 1 = 0$ i.e. $lambda^2 = 1.$ So $lambda = pm 1.$



      So according to my argument above it follows that eigenvalues of a real orthogonal matrix are $pm 1.$ But I think that I am wrong as I know that the eigenvalues of an orthogonal matrix are unit modulus i.e. they lie on the unit circle.



      What's going wrong in my argument above. Please help me in this regard.



      Thank you very much for your valuable time.










      share|cite|improve this question











      $endgroup$




      Let $A$ be a real orthogonal matrix. Then $A^text T A = I.$ Let $lambda in Bbb C$ be an eigenvalue of $A$ corresponding to the eigenvector $X in Bbb C^n.$ Then we have





      $$beginalign* X^text T A^text T A X = X^text T X. \ implies (AX)^text T AX & = X^text T X. \ implies (lambda X)^text T lambda X & = X^text T X. \ implies lambda^2 X^text T X & = X^text T X. \ implies (lambda^2 - 1) X^text T X & = 0. endalign*$$





      Since $X$ is an eigenvector $X neq 0.$ Therefore $_2^2 = X^text T X neq 0.$ Hence we must have $lambda^2 - 1 = 0$ i.e. $lambda^2 = 1.$ So $lambda = pm 1.$



      So according to my argument above it follows that eigenvalues of a real orthogonal matrix are $pm 1.$ But I think that I am wrong as I know that the eigenvalues of an orthogonal matrix are unit modulus i.e. they lie on the unit circle.



      What's going wrong in my argument above. Please help me in this regard.



      Thank you very much for your valuable time.







      linear-algebra eigenvalues-eigenvectors orthogonal-matrices






      share|cite|improve this question















      share|cite|improve this question













      share|cite|improve this question




      share|cite|improve this question








      edited yesterday









      Yanko

      8,2672830




      8,2672830










      asked 2 days ago









      math maniac.math maniac.

      1417




      1417




















          1 Answer
          1






          active

          oldest

          votes


















          4












          $begingroup$

          The mistake is your assumption that $X^TXne0$. Consider a simple example:
          $$A=pmatrix0&1\-1&0.$$
          It is orthogonal, and its eigenvalues are $pm i$. One eigenvector is
          $$X=pmatrix1\i.$$
          It satisfies $X^TX=0$.



          However, replacing $X^T$ in your argument by $X^H$ (complex conjugate
          of transpose) will give you the correct conclusion that $|lambda|^2=1$.






          share|cite|improve this answer









          $endgroup$












          • $begingroup$
            how can Euclidean norm of non zero vector be zero?
            $endgroup$
            – math maniac.
            2 days ago






          • 2




            $begingroup$
            @mathmaniac. How can $1^2+i^2$ equal zero?
            $endgroup$
            – Lord Shark the Unknown
            2 days ago










          • $begingroup$
            I think the Euclidean norm of $X in Bbb C^n$ is $sqrt X^text T overline X text or sqrt overline X^text T X,$ not $sqrt X^text T X.$ Am I right?
            $endgroup$
            – math maniac.
            2 days ago







          • 1




            $begingroup$
            Which is same as $sqrt X^text HX,$ as you have rightly pointed out.
            $endgroup$
            – math maniac.
            yesterday












          Your Answer





          StackExchange.ifUsing("editor", function ()
          return StackExchange.using("mathjaxEditing", function ()
          StackExchange.MarkdownEditor.creationCallbacks.add(function (editor, postfix)
          StackExchange.mathjaxEditing.prepareWmdForMathJax(editor, postfix, [["$", "$"], ["\\(","\\)"]]);
          );
          );
          , "mathjax-editing");

          StackExchange.ready(function()
          var channelOptions =
          tags: "".split(" "),
          id: "69"
          ;
          initTagRenderer("".split(" "), "".split(" "), channelOptions);

          StackExchange.using("externalEditor", function()
          // Have to fire editor after snippets, if snippets enabled
          if (StackExchange.settings.snippets.snippetsEnabled)
          StackExchange.using("snippets", function()
          createEditor();
          );

          else
          createEditor();

          );

          function createEditor()
          StackExchange.prepareEditor(
          heartbeatType: 'answer',
          autoActivateHeartbeat: false,
          convertImagesToLinks: true,
          noModals: true,
          showLowRepImageUploadWarning: true,
          reputationToPostImages: 10,
          bindNavPrevention: true,
          postfix: "",
          imageUploader:
          brandingHtml: "Powered by u003ca class="icon-imgur-white" href="https://imgur.com/"u003eu003c/au003e",
          contentPolicyHtml: "User contributions licensed under u003ca href="https://creativecommons.org/licenses/by-sa/3.0/"u003ecc by-sa 3.0 with attribution requiredu003c/au003e u003ca href="https://stackoverflow.com/legal/content-policy"u003e(content policy)u003c/au003e",
          allowUrls: true
          ,
          noCode: true, onDemand: true,
          discardSelector: ".discard-answer"
          ,immediatelyShowMarkdownHelp:true
          );



          );













          draft saved

          draft discarded


















          StackExchange.ready(
          function ()
          StackExchange.openid.initPostLogin('.new-post-login', 'https%3a%2f%2fmath.stackexchange.com%2fquestions%2f3169070%2feigenvalues-of-a-real-orthogonal-matrix%23new-answer', 'question_page');

          );

          Post as a guest















          Required, but never shown

























          1 Answer
          1






          active

          oldest

          votes








          1 Answer
          1






          active

          oldest

          votes









          active

          oldest

          votes






          active

          oldest

          votes









          4












          $begingroup$

          The mistake is your assumption that $X^TXne0$. Consider a simple example:
          $$A=pmatrix0&1\-1&0.$$
          It is orthogonal, and its eigenvalues are $pm i$. One eigenvector is
          $$X=pmatrix1\i.$$
          It satisfies $X^TX=0$.



          However, replacing $X^T$ in your argument by $X^H$ (complex conjugate
          of transpose) will give you the correct conclusion that $|lambda|^2=1$.






          share|cite|improve this answer









          $endgroup$












          • $begingroup$
            how can Euclidean norm of non zero vector be zero?
            $endgroup$
            – math maniac.
            2 days ago






          • 2




            $begingroup$
            @mathmaniac. How can $1^2+i^2$ equal zero?
            $endgroup$
            – Lord Shark the Unknown
            2 days ago










          • $begingroup$
            I think the Euclidean norm of $X in Bbb C^n$ is $sqrt X^text T overline X text or sqrt overline X^text T X,$ not $sqrt X^text T X.$ Am I right?
            $endgroup$
            – math maniac.
            2 days ago







          • 1




            $begingroup$
            Which is same as $sqrt X^text HX,$ as you have rightly pointed out.
            $endgroup$
            – math maniac.
            yesterday
















          4












          $begingroup$

          The mistake is your assumption that $X^TXne0$. Consider a simple example:
          $$A=pmatrix0&1\-1&0.$$
          It is orthogonal, and its eigenvalues are $pm i$. One eigenvector is
          $$X=pmatrix1\i.$$
          It satisfies $X^TX=0$.



          However, replacing $X^T$ in your argument by $X^H$ (complex conjugate
          of transpose) will give you the correct conclusion that $|lambda|^2=1$.






          share|cite|improve this answer









          $endgroup$












          • $begingroup$
            how can Euclidean norm of non zero vector be zero?
            $endgroup$
            – math maniac.
            2 days ago






          • 2




            $begingroup$
            @mathmaniac. How can $1^2+i^2$ equal zero?
            $endgroup$
            – Lord Shark the Unknown
            2 days ago










          • $begingroup$
            I think the Euclidean norm of $X in Bbb C^n$ is $sqrt X^text T overline X text or sqrt overline X^text T X,$ not $sqrt X^text T X.$ Am I right?
            $endgroup$
            – math maniac.
            2 days ago







          • 1




            $begingroup$
            Which is same as $sqrt X^text HX,$ as you have rightly pointed out.
            $endgroup$
            – math maniac.
            yesterday














          4












          4








          4





          $begingroup$

          The mistake is your assumption that $X^TXne0$. Consider a simple example:
          $$A=pmatrix0&1\-1&0.$$
          It is orthogonal, and its eigenvalues are $pm i$. One eigenvector is
          $$X=pmatrix1\i.$$
          It satisfies $X^TX=0$.



          However, replacing $X^T$ in your argument by $X^H$ (complex conjugate
          of transpose) will give you the correct conclusion that $|lambda|^2=1$.






          share|cite|improve this answer









          $endgroup$



          The mistake is your assumption that $X^TXne0$. Consider a simple example:
          $$A=pmatrix0&1\-1&0.$$
          It is orthogonal, and its eigenvalues are $pm i$. One eigenvector is
          $$X=pmatrix1\i.$$
          It satisfies $X^TX=0$.



          However, replacing $X^T$ in your argument by $X^H$ (complex conjugate
          of transpose) will give you the correct conclusion that $|lambda|^2=1$.







          share|cite|improve this answer












          share|cite|improve this answer



          share|cite|improve this answer










          answered 2 days ago









          Lord Shark the UnknownLord Shark the Unknown

          107k1162135




          107k1162135











          • $begingroup$
            how can Euclidean norm of non zero vector be zero?
            $endgroup$
            – math maniac.
            2 days ago






          • 2




            $begingroup$
            @mathmaniac. How can $1^2+i^2$ equal zero?
            $endgroup$
            – Lord Shark the Unknown
            2 days ago










          • $begingroup$
            I think the Euclidean norm of $X in Bbb C^n$ is $sqrt X^text T overline X text or sqrt overline X^text T X,$ not $sqrt X^text T X.$ Am I right?
            $endgroup$
            – math maniac.
            2 days ago







          • 1




            $begingroup$
            Which is same as $sqrt X^text HX,$ as you have rightly pointed out.
            $endgroup$
            – math maniac.
            yesterday

















          • $begingroup$
            how can Euclidean norm of non zero vector be zero?
            $endgroup$
            – math maniac.
            2 days ago






          • 2




            $begingroup$
            @mathmaniac. How can $1^2+i^2$ equal zero?
            $endgroup$
            – Lord Shark the Unknown
            2 days ago










          • $begingroup$
            I think the Euclidean norm of $X in Bbb C^n$ is $sqrt X^text T overline X text or sqrt overline X^text T X,$ not $sqrt X^text T X.$ Am I right?
            $endgroup$
            – math maniac.
            2 days ago







          • 1




            $begingroup$
            Which is same as $sqrt X^text HX,$ as you have rightly pointed out.
            $endgroup$
            – math maniac.
            yesterday
















          $begingroup$
          how can Euclidean norm of non zero vector be zero?
          $endgroup$
          – math maniac.
          2 days ago




          $begingroup$
          how can Euclidean norm of non zero vector be zero?
          $endgroup$
          – math maniac.
          2 days ago




          2




          2




          $begingroup$
          @mathmaniac. How can $1^2+i^2$ equal zero?
          $endgroup$
          – Lord Shark the Unknown
          2 days ago




          $begingroup$
          @mathmaniac. How can $1^2+i^2$ equal zero?
          $endgroup$
          – Lord Shark the Unknown
          2 days ago












          $begingroup$
          I think the Euclidean norm of $X in Bbb C^n$ is $sqrt X^text T overline X text or sqrt overline X^text T X,$ not $sqrt X^text T X.$ Am I right?
          $endgroup$
          – math maniac.
          2 days ago





          $begingroup$
          I think the Euclidean norm of $X in Bbb C^n$ is $sqrt X^text T overline X text or sqrt overline X^text T X,$ not $sqrt X^text T X.$ Am I right?
          $endgroup$
          – math maniac.
          2 days ago





          1




          1




          $begingroup$
          Which is same as $sqrt X^text HX,$ as you have rightly pointed out.
          $endgroup$
          – math maniac.
          yesterday





          $begingroup$
          Which is same as $sqrt X^text HX,$ as you have rightly pointed out.
          $endgroup$
          – math maniac.
          yesterday


















          draft saved

          draft discarded
















































          Thanks for contributing an answer to Mathematics Stack Exchange!


          • Please be sure to answer the question. Provide details and share your research!

          But avoid


          • Asking for help, clarification, or responding to other answers.

          • Making statements based on opinion; back them up with references or personal experience.

          Use MathJax to format equations. MathJax reference.


          To learn more, see our tips on writing great answers.




          draft saved


          draft discarded














          StackExchange.ready(
          function ()
          StackExchange.openid.initPostLogin('.new-post-login', 'https%3a%2f%2fmath.stackexchange.com%2fquestions%2f3169070%2feigenvalues-of-a-real-orthogonal-matrix%23new-answer', 'question_page');

          );

          Post as a guest















          Required, but never shown





















































          Required, but never shown














          Required, but never shown












          Required, but never shown







          Required, but never shown

































          Required, but never shown














          Required, but never shown












          Required, but never shown







          Required, but never shown







          Popular posts from this blog

          Wikipedia:Vital articles Мазмуну Biography - Өмүр баян Philosophy and psychology - Философия жана психология Religion - Дин Social sciences - Коомдук илимдер Language and literature - Тил жана адабият Science - Илим Technology - Технология Arts and recreation - Искусство жана эс алуу History and geography - Тарых жана география Навигация менюсу

          Bruxelas-Capital Índice Historia | Composición | Situación lingüística | Clima | Cidades irmandadas | Notas | Véxase tamén | Menú de navegacióneO uso das linguas en Bruxelas e a situación do neerlandés"Rexión de Bruxelas Capital"o orixinalSitio da rexiónPáxina de Bruselas no sitio da Oficina de Promoción Turística de Valonia e BruxelasMapa Interactivo da Rexión de Bruxelas-CapitaleeWorldCat332144929079854441105155190212ID28008674080552-90000 0001 0666 3698n94104302ID540940339365017018237

          What should I write in an apology letter, since I have decided not to join a company after accepting an offer letterShould I keep looking after accepting a job offer?What should I do when I've been verbally told I would get an offer letter, but still haven't gotten one after 4 weeks?Do I accept an offer from a company that I am not likely to join?New job hasn't confirmed starting date and I want to give current employer as much notice as possibleHow should I address my manager in my resignation letter?HR delayed background verification, now jobless as resignedNo email communication after accepting a formal written offer. How should I phrase the call?What should I do if after receiving a verbal offer letter I am informed that my written job offer is put on hold due to some internal issues?Should I inform the current employer that I am about to resign within 1-2 weeks since I have signed the offer letter and waiting for visa?What company will do, if I send their offer letter to another company